2
$\begingroup$

I am interested in the long term behaviour of the sequence $a_{n+1}=a_0^{a_n}$ where $a_0\in\mathbb{C}$. I am aware there are many sources discussing convergence of this sequence(for example) but I am interested in the long-term oscillatory behaviour of the sequence which I cannot find any discussion on. I will mention I am using the principle branch of the logarithm. Power tower regions of "convergence" Above is a plot I have made displaying what period the sequences approach up to period $25$(any higher period is plotted as divergent). I have noticed several properties of these regions of "convergence" although have no idea how I would go about proving them.

Firstly it appears that if the sequence approaches a cycle of $m$ numbers $\{\xi_1,\xi_2\cdots,\xi_m\}$ we can find $i,j$ such that the sequences defined by $a_0=\xi_i,a_0=\xi_j$ converge and have period-$2$. For example in the picture the blue point defines a sequence which approaches period $4$ and there two numbers in this cycle which specify convergent and period-$2$ sequences. One can make a more general statement about $m$ cycles containing numbers specifying sequences with cycles up to $m-1$ or similar however I have not been able to check this well and so am less sure about the truth of this claim.

Secondly, I would like to know if one can give details about certain regions of low period. For example one can easily show if $\Re(a_0)<0$ as $|a_0|\to\infty$ then the sequence approaches the cycle $\{a_0,0,1\}$ as if $a_0=Re^{i\theta}, a_1=Re^{i\theta R(\cos(\theta)+i\sin(\theta))}=Re^{i\theta\cos(\theta)}e^{-R\theta\sin(\theta)}\approx0$. I was wondering if this can be strengthened to say that if $|a_0|$ is sufficiently large then you can guarantee that it approaches a period-$3$ cycle. I would also be interested if one can specify the region of convergence and region around $0$ which approaches a period-$2$ cycle in terms of $a_0$. One would maybe guess that for $|a_0|<e^{-e}$ the sequence approaches a period-$2$ cycle which appears true although it appears that this does not full specify the region.

The only other thing I noticed is that for all $n$ one appears to be able to find and $a_0$ such that the sequence approaches a cycle of period $n$.

I apologise if any of the above has already been asked and would appreciate answers to any of them and any other observations I missed are greatly welcomed.

$\endgroup$
3
  • 1
    $\begingroup$ I am aware there are many sources discussing convergence of this sequence(for example) but I am interested in the long-term oscillatory behaviour of the sequence which I cannot find any discussion on. -- I imagine you've come across Ioannis Galidakis's list of references by now (they've been online since 2003 or 2004), but if not, then (if you're not already aware of them) maybe look at #6, 7, 26, 41, 45, 46 here. Perhaps also some of Galidakis's publications, and also what google-scholar (and MathSciNet and Zbl) searches produce for appropriate authors and terms. $\endgroup$ Commented Nov 10 at 17:13
  • $\begingroup$ You seem to be observing some features similar to those guaranteed by Sharkovskii's theorem, although that theorem only applies to functions on a real interval. $\endgroup$ Commented Nov 11 at 19:52
  • $\begingroup$ @DaveL.Renfro Thank you I found the answers I was looking for in this paper $\endgroup$ Commented Nov 12 at 10:55

0

You must log in to answer this question.

Start asking to get answers

Find the answer to your question by asking.

Ask question

Explore related questions

See similar questions with these tags.